Tải bản đầy đủ (.pdf) (11 trang)

Các định lý và bổ đề trong số học - Lê Phúc Lữ

Bạn đang xem bản rút gọn của tài liệu. Xem và tải ngay bản đầy đủ của tài liệu tại đây (282.93 KB, 11 trang )

<span class='text_page_counter'>(1)</span><div class='page_container' data-page=1>

<b>Chủ đề 4. </b>


<b>CÁC ĐỊNH LÝ, BỔ ĐỀ TRONG SỐ HỌC </b>


<b>VÀ ỨNG DỤNG </b>



<b>I) Giới thiệu. </b>


<i>Trong nội dung này, ta sẽ việc áp dụng của các định lý, bổ đề trong một số bài toán số học, tổ </i>
<i>hợp. Cụ thể là: </i>


<i>- Các đánh giá số mũ đúng và bổ đề LTE. </i>


<i>Với p là số nguyên tố lẻ và a b</i>, <i> thỏa mãn a b</i>, <i> khơng chia hết cho p</i>,<i> cịn a b</i> <i> chia hết </i>
<i>cho p thì v a<sub>p</sub></i>( <i>n</i><i>bn</i>)<i>v a b<sub>p</sub></i>(  )<i>v n<sub>p</sub></i>( )<i> với mọi n</i><i>. </i>


<i>Đặc biệt nếu p</i>2<i> thì ta cần điều kiện </i>4 |<i>a b</i> <i>. </i>
<i>- Tính chất: </i> gcd( , )


gcd( <i>m</i> 1, <i>n</i> 1) <i>m n</i> 1


<i>a</i>  <i>a</i>  <i>a</i>  <i> với a</i><i> và m n</i>, 


 <i>. Nếu như m n</i>, <i> lẻ, ta có thể </i>
<i>thay a bởi </i><i>a để được </i> gcd( , )


gcd(<i>am</i>1,<i>an</i>1)<i>a</i> <i>m n</i> 1<i>. </i>


<i>- Tính chất: </i>gcd , gcd( , )
<i>n</i> <i>n</i>


<i>a</i> <i>b</i>



<i>a b</i> <i>a b n</i>


<i>a b</i>


  


  


 




 


<i> với mọi a b n</i>, , 
 <i>. </i>
<i>- Cơng thức tính tổng số ước, số các số ước: </i>


<i>Cho </i> 1 2


1 2
<i>k</i>
<i>a</i>
<i>a</i> <i>a</i>


<i>k</i>


<i>n</i> <i>p p</i> <i>p</i> <i> với p p</i>1, 2,,<i>pk là các số nguyên tố và các số mũ a a</i>1, 2,,<i>ak nguyên </i>
<i>dương. Khi đó </i>



<i>+ Tổng số ước của n là </i> ( ) (1 <i>ai</i>)


<i>i</i> <i>i</i>


<i>n</i> <i>p</i> <i>p</i>


 

<sub></sub>

  <i> với tích lấy trên các ước của n</i>.<i> </i>
<i>+ Số các ước của n là </i>( )<i>n</i> (1<i>a</i><sub>1</sub>)(1<i>a</i><sub>2</sub>)(1<i>a<sub>k</sub></i>).<i> </i>


<b>II) Các bài tập áp dụng. </b>


<b>Bài 1. </b>a) Cho các số nguyên dương <i>a b c</i>, , thỏa mãn gcd( , , ) 1<i>a b c</i>  và <i>a bc b ca c ab</i>| , | , | . Chứng
minh rằng <i>bc</i>


<i>a</i> là số chính phương.


b) Tìm giá trị lớn nhất của 2


2( 6 17)


<i>v n</i>  <i>n</i> với <i>n</i> là số nguyên dương.


<b>Lời giải. </b>


a) Ta có <i>ab</i> <i>abc</i><sub>2</sub>


<i>c</i>  <i>c</i> nên cần chứng minh <i>abc</i> là số chính phương. Xét số nguyên tố <i>p abc</i>| . Khi


</div>
<span class='text_page_counter'>(2)</span><div class='page_container' data-page=2>

( ) ( ) ( ) ( )


( ) ( ) ( ) ( )


<i>p</i> <i>p</i> <i>p</i> <i>p</i>


<i>p</i> <i>p</i> <i>p</i> <i>p</i>


<i>v ab</i> <i>v c</i> <i>v b</i> <i>v c</i>


<i>v ac</i> <i>v b</i> <i>v c</i> <i>v b</i>


 


 


 




 


 


 


 


.


b) Ta có 2 2
6 17 ( 3) 8



<i>n</i>  <i>n</i>  <i>n</i>  nên 2

<sub></sub>

2

<sub></sub>


2( 6 17) min 2( 3) , 28 3.


<i>v n</i>  <i>n</i>  <i>v n</i> <i>v</i>  Điều này có
được là do 2


2( 3)


<i>v n</i> luôn là số chẵn.


Đến đây chọn <i>n</i>1 thì ta có đẳng thức xảy ra.


<b>Bài 2. </b>a) Tìm số nguyên dương <i>n</i> nhỏ nhất sao cho


10
10


7 | 2 1


5 | 3 1


<i>n</i>
<i>n</i>


 











.


b) Cho ( ) 2016<i>n</i> 1


<i>f n</i>   với <i>n</i>. Chứng minh rằng tồn tại vô hạn số nguyên dương <i>a</i> sao
cho dãy số xác định bởi <i>u</i>0 <i>a</i> và <i>un</i>1  <i>f u</i>( <i>n</i>) có tất cả các số hạng đều chia hết cho <i>a</i>.


c) Cho <i>a n</i>, là các số nguyên dương và <i>p</i> là số nguyên tố lẻ thỏa mãn <i>ap</i> 1(mod<i>pn</i>), chứng
minh rằng 1


1(mod <i>n</i> ).


<i>a</i> <i>p</i> 


<b>Lời giải. </b>


a) Theo giả thiết thì ta phải có 3 |<i>n</i> và 4 |<i>n</i>. Thật vậy,
Đặt <i>n</i>3<i>k</i><i>r</i> với <i>k</i> và <i>r</i>

0,1, 2

thì


3


2<i>n</i> 1 2<i>k r</i> 1 8<i>k</i> 2<i>r</i> 1 2<i>r</i> 1(mod 7)


        .



Thử các giá trị thì chỉ có <i>r</i>0 thỏa mãn.
Chứng minh tương tự để có 4 | .<i>n</i>


Đến đây dùng định lý LTE, ta chỉ ra được rằng


7


3 ( ) 9


<i>n</i> <i>a</i><i>v a</i>  và <i>n</i>4<i>b</i><i>v b</i><sub>5</sub>( )9.
Do đó 9


12 35
<i>n</i>  .


b) Chọn <i>a</i>2017<i>m</i> với <i>m</i> ngun dương, sau đó tính <i>v</i>2017 của các số hạng và chứng tỏ rằng


tất cả các số dạng này thỏa mãn đề bài.


c) Theo giả thiết thì ( , ) 1<i>a p</i>  nên theo định lý Fermat nhỏ thì 1


1(mod ).
<i>p</i>


<i>a</i>   <i>p</i> Suy ra


1


(mod )
<i>p</i> <i>p</i>



<i>a</i> <i>a</i>  <i>p</i> hay <i>a</i>1(mod ).<i>p</i>


Đến đây, áp dụng LTE


( <i>p</i> 1) ( 1) ( ) ( 1) 1.


<i>p</i> <i>p</i> <i>p</i> <i>p</i>


</div>
<span class='text_page_counter'>(3)</span><div class='page_container' data-page=3>

<b>Bài 3. (PTNK 2015) </b>Cho <i>f</i> :  thỏa mãn <i>f n</i>( ) 2


<i>f</i> <i>n</i>


<i>n</i>


 




 


  với mọi <i>n</i> nguyên dương. Xét
<i>n</i> nguyên dương lớn hơn 1 và đặt <i>a</i><i>v n b<sub>p</sub></i>( ), <i>v<sub>p</sub></i>

<i>f n</i>( )

với số nguyên tố <i>p a</i>| .


a) Chứng minh rằng 1 <i>b</i> 3


<i>a</i>


  và 5 11.



3 5


<i>b</i>
<i>a</i>


 
b) Tìm tất cả các hàm số <i>f n</i>( ) thỏa mãn đề bài.


<b>Lời giải. </b>


a) Theo giả thiết thì <i>n f n</i>| ( ) nên <i>b</i><i>a</i>.
Thay <i>n</i> <i>f n</i>( )


<i>n</i>


 trong điều kiện <i>n f n</i>| ( ), ta có <i>f n</i>( ) 2
<i>n</i>


<i>n</i> hay <i>b</i>3 .<i>a</i> Một cách tương tự ta có


đánh giá thứ hai.


b) Đánh giá ở câu b có thể chứng minh bằng quy nạp rằng 2 1 2 2


2 2 1


<i>n</i> <i>n</i>


<i>n</i> <i>n</i>



<i>u</i> <i>b</i> <i>u</i>


<i>u</i> <i>a</i> <i>u</i>


 




  trong đó


1 2


2 1


1, 3,
2


<i>n</i> <i>n</i> <i>n</i>


<i>u</i> <i>u</i>


<i>u</i> <sub></sub> <i>u</i> <sub></sub> <i>u</i>


 





 





.


Dãy số này có cơng thức tổng qt là


1
2 ( 1)


3


<i>n</i> <i>n</i>


<i>n</i>
<i>u</i>



 


 nên dễ dàng có được


2 1 2 2


2 2 1


lim <i>n</i> lim <i>n</i> 2


<i>n</i> <i>n</i>


<i>u</i> <i>u</i>



<i>u</i> <i>u</i>


 




  .


Suy ra <i>b</i>2<i>a</i> nên 2
( )


<i>f n</i> <i>n</i> với <i>n</i>2.


Trường hợp <i>n</i>1, giả sử <i>f</i>(1)<i>a</i>1 thì thay <i>n</i>1 vào đề bài, ta có


( ) 1


<i>f a</i>  , mâu thuẫn vì đã có 2
( ) 1.
<i>f a</i> <i>a</i> 
Do đó, <i>f</i>(1) 1 và 2


( )


<i>f n</i> <i>n</i> với mọi <i>n</i>.
Thử lại ta thấy thỏa.


<b>Bài 4. </b>Số nguyên dương <i>n</i> gọi là tốt nếu như | 2<i>n</i> 1.



<i>n</i> 


a) Gọi <i>s n</i>( ) là số các số tốt không vượt quá <i>n</i>. Chứng minh rằng <i>s n</i>( )log<sub>3</sub><i>n</i>.
b) Giả sử <i>x y</i>, là các số tốt. Chứng minh rằng gcd( , )<i>x y</i> và lcm( , )<i>x y</i> cũng tốt.


</div>
<span class='text_page_counter'>(4)</span><div class='page_container' data-page=4>

d) Chứng minh rằng tồn tại số tốt có đúng 2000 ước nguyên tố.


<b>Lời giải.</b>


a) Xét các số lũy thừa của 3. Chú ý rằng nếu đặt <i>k</i> thỏa mãn 1


3<i>k</i> <i>n</i>3<i>k</i> thì log ( )3 <i>n</i> <i>k</i>1 và


có ít nhất <i>k</i>1 số tốt không vượt quá <i>n</i>.
b) Chú ý bổ đề gcd( , )


gcd(2<i>a</i>1, 2<i>b</i>1)2 <i>a b</i> 1 với <i>a b</i>, là các số lẻ.
Ý lcm là khá hiển nhiên, cịn ý gcd thì dùng bổ đề trên.


c) Vì | 2<i>n</i> 1


<i>n</i>  nên đặt 2<i>n</i> 1 <i>kn</i> với <i>k</i> lẻ thì 2 1


2 <i>n</i> 2<i>kn</i>1 chia hết cho 2<i>n</i>1. Ở ý <i>np</i>, ta sử
dụng <i>v<sub>p</sub></i> thích hợp và chia thành hai trường hợp: nếu <i>p n</i>| và nếu <i>p n</i>| .


d) Đây là đề IMO 2000. Ý tưởng giải chính là dùng câu c, khi ta chọn ước nguyên tố của 2<i>x</i>1


mà lớn hơn <i>x</i> (với <i>x</i> là số tốt) rồi ghép với <i>x</i> thì số lượng ước nguyên tố của <i>x</i> tăng lên 1.



<b>Bài 5. </b>Cho biết rằng với <i>n</i> nguyên dương thì ( !) ( )
1


<i>p</i>
<i>p</i>


<i>n s n</i>
<i>v n</i>


<i>p</i>



 trong đó <i>s np</i>( ) là tổng các chữ


số của <i>n</i> trong hệ <i>p</i>- phân; hãy giải các bài toán sau.


a) Cho <i>a b c</i>, , là các số nguyên dương và <i>a b c</i>! !| !. Chứng minh rằng


2 2
2<i>a b c</i>   <i>c</i> .
b) Với <i>n</i> là số nguyên dương chẵn, đặt


1 1 1


1!( 1)! 3!( 3)! ( 1)!1!
<i>n</i>


<i>a</i>



<i>n</i> <i>n</i> <i>n</i>


   


    .


Tìm tất cả các số nguyên dương <i>n</i> sao cho 2<i>x</i> <i>an</i>(2<i>y</i>1) có nghiệm nguyên dương ( ; ).<i>x y</i>


<b>Gợi ý. </b>


a) Tính <i>v</i><sub>2</sub> hai vế <i>a b c</i>! !| !, ta có


2( !) 2( !) 2( !)


<i>v a</i> <i>v b</i> <i>v c</i> hay <i>a</i><i>s a</i>2( ) <i>b s b</i>2( ) <i>c</i> <i>s c</i>2( ).


Với mọi <i>n</i>, ta có <i>s n</i><sub>2</sub>( )log ( ) 1.<sub>2</sub> <i>n</i>  Suy ra


2


2 2 2 2 2 2


2 ( ) ( ) ( ) 2 log ( ) log ( ) log ( ).


<i>a</i>   <i>b c</i> <i>s a</i> <i>s b</i> <i>s c</i>   <i>a</i>  <i>b</i>  <i>c</i>
Do đó 2 2


2<i>a b c</i>   <i>c</i> .


b) Thu gọn biểu thức đã cho, ta được



1
2


!
<i>n</i>
<i>n</i>
<i>a</i>


</div>
<span class='text_page_counter'>(5)</span><div class='page_container' data-page=5>

Dùng hàm định giá <i>v n</i><sub>2</sub>( !)<i>n s n</i> <sub>2</sub>( ), ta thấy tất cả các số cần tìm là <i>n</i> chẵn, <i>n</i>6 và <i>n</i> không
phải là lũy thừa của 2.


<b>Bài 6. (USA MTS 2015) </b>Cho số nguyên dương <i>n</i>3. Người ta xếp <i>n</i> số nguyên dương nào đó
trên một đường trịn sao cho:


i. Tích của hai số khơng nằm cạnh nhau thì chia hết cho 2015 2016 .
ii. Tích của hai số nằm cạnh nhau thì khơng chia hết cho 2015 2016 .
Tìm giá trị lớn nhất của <i>n</i>.


<b>Lời giải. </b>


Ta có 5 2


2015 5 13 31, 2016 2 3 7  . Đặt <i>M</i> 2015 2016 và <i>N</i> 

2, 3, 5, 7,13, 31

.


Giả sử tồn tại <i>n</i> số nguyên dương <i>x x x</i><sub>1</sub>, <sub>2</sub>, <sub>3</sub>,...,<i>x<sub>n</sub></i> xếp trên vòng trịn thỏa mãn tính chất đã
nêu, quy ước <i>x<sub>n</sub></i><sub></sub><sub>1</sub><i>x</i><sub>1</sub> và <i>x<sub>n</sub></i><sub></sub><sub>2</sub> <i>x</i><sub>2</sub>. Do <i>x x<sub>i</sub></i> <i><sub>i</sub></i><sub></sub><sub>1</sub> không chia hết cho <i>M</i> với <i>i</i>1,<i>n</i> nên tồn tại số
nguyên tố <i>p<sub>i</sub></i><i>N</i> tương ứng mà


1 1



( ) ( ) ( ) ( ) ( )


<i>i</i> <i>i</i> <i>i</i> <i>i</i> <i>i</i>


<i>p</i> <i>i</i> <i>i</i> <i>p</i> <i>p</i> <i>i</i> <i>p</i> <i>i</i> <i>p</i>


<i>v</i> <i>x x</i><sub></sub> <i>v</i> <i>M</i> <i>v</i> <i>x</i> <i>v</i> <i>x</i><sub></sub> <i>v</i> <i>M</i> (*)


Chú ý rằng các số <i>p<sub>i</sub></i> này có thể trùng nhau. Ta sẽ chứng minh rằng <i>p<sub>i</sub></i>  <i>p<sub>j</sub></i> với <i>i j</i>, là hai chỉ
số không kề nhau.


Thật vậy, giả sử tồn tại <i>i j</i>, không kề nhau sao cho <i>pi</i>  <i>pj</i>  <i>p</i> (do <i>n</i>4 nên tồn tại các chỉ


số <i>i j</i>, như thế). Đặt <i>a</i><i>v M<sub>p</sub></i>( ) thì theo (*), ta có


1


( ) ( )


<i>p</i> <i>i</i> <i>p</i> <i>i</i>


<i>v</i> <i>x</i> <i>v</i> <i>x</i> <i>a</i> và <i>vp</i>(<i>xj</i>)<i>vp</i>(<i>xj</i>1)<i>a</i>.


Suy ra <i>v<sub>p</sub></i>( )<i>x<sub>i</sub></i> <i>v<sub>p</sub></i>(<i>x<sub>i</sub></i><sub></sub><sub>1</sub>)<i>v<sub>p</sub></i>(<i>x<sub>j</sub></i>)<i>v<sub>p</sub></i>(<i>x<sub>j</sub></i><sub></sub><sub>1</sub>)2<i>a</i> (1)
Mặt khác, do ( , ), (<i>i j</i> <i>i</i>1, <i>j</i>1) là các cặp số không kề nhau nên


1 1


( ) ( ) , ( ) ( )



<i>p</i> <i>i</i> <i>p</i> <i>j</i> <i>p</i> <i>i</i> <i>p</i> <i>j</i>


<i>v</i> <i>x</i> <i>v</i> <i>x</i> <i>a v</i> <i>x</i><sub></sub> <i>v</i> <i>x</i> <sub></sub> <i>a</i>.
Suy ra <i>vp</i>( )<i>xi</i> <i>vp</i>(<i>xi</i>1)<i>vp</i>(<i>xj</i>)<i>vp</i>(<i>xj</i>1)2<i>a</i> (2)


Ta thấy (1), (2) mâu thuẫn nhau nên nhận xét trên được chứng minh.


Tiếp theo, giả sử tồn tại <i>i</i> mà <i>pi</i>  <i>pi</i>1  <i>p</i> (với các số <i>p pi</i>, <i>j</i> định nghĩa như trên). Nếu


( ) 1


<i>p</i>


<i>v M</i>  thì do các cặp <i>x xi</i> <i>i</i>1,<i>x xi</i>1 <i>i</i>2 đều không chia hết cho <i>M</i> nên dẫn đến <i>x xi</i>, <i>i</i>1,<i>xi</i>2 đều


</div>
<span class='text_page_counter'>(6)</span><div class='page_container' data-page=6>

Từ đây suy ra, trong dãy các số <i>p p</i><sub>1</sub>, <sub>2</sub>,...,<i>p<sub>n</sub></i> với <i>p<sub>i</sub></i> <i>N</i>, mỗi số nguyên tố xuất hiện khơng
q 2 lần và nếu nó xuất hiện 2 lần thì bình phương của nó phải là ước của <i>M</i> . Nói cách khác,


<i>M</i> chia hết cho tích <i>p p</i><sub>1</sub> <sub>2</sub>...<i>p<sub>n</sub></i>.
Do 5 2


2 3 5 7 13 31


<i>M</i>       nên để cho số lượng

<i>n</i>

lớn nhất có thể, ta cho số 2 xuất hiện 2 lần,
số 3 xuất hiện 2 lần, các ước nguyên tố cịn lại xuất hiện 1 lần, tức là có tổng cộng 8 số.
Ta lại có thể chọn các số như sau: <sub>5</sub> <sub>2</sub>


31



, , , , , , ,


2 2 3 3 3 5 5 7 7 13 13 31 2


<i>M</i> <i>M</i> <i>M</i> <i>M</i> <i>M</i> <i>M</i> <i>M</i> <i>M</i>




     (tương ứng với


dãy số nguyên tố<i>p</i><sub>1</sub>2,<i>p</i><sub>2</sub> 3,<i>p</i><sub>3</sub> 3,<i>p</i><sub>4</sub> 5,<i>p</i><sub>5</sub> 7,<i>p</i><sub>6</sub> 13,<i>p</i><sub>7</sub> 31,<i>p</i><sub>8</sub> 2).


Rõ ràng dãy số này thỏa mãn các điều kiện đã cho. Vậy giá trị lớn nhất của <i>n</i> là 8.


<b>Bài 7. </b>Xâu nhị phân là một dãy liên tiếp các ký tự 0,1 được xếp thành một hàng, cịn bậc của
nó là số xâu nhị phân khơng trùng với nó nhưng có cùng số lượng ký tự 0,1 với nó.


<i>Ví dụ: xâu nhị phân </i>001<i> có bậc </i>2<i> vì có hai xâu nhị phân </i>100, 010<i> có cùng số lượng </i>0,1<i> với nó. </i>


Hỏi có bao nhiêu cách xếp 66 số 0 và 33 số 1 thành một dãy sao cho với mọi <i>k</i>1,99 thì
xâu nhị phân tính từ vị trí thứ 1 đến vị trí thứ <i>k</i> trong dãy đều có bậc chẵn?


<b>Lời giải. </b>Ta sử dụng bổ đề sau (suy ra từ định lý Lucas):


Cho hai số nguyên dương <i>m n</i>, với <i>m</i><i>n</i> có các biểu diễn nhị phân là


1 2 <i>k</i>


<i>m</i><i>m m</i> <i>m</i> và <i>n</i><i>n n</i>1 2<i>nk</i> .



Khi đó nếu tồn tại <i>i</i>

1, 2, 3,,<i>k</i>

mà <i>m<sub>i</sub></i> <i>n<sub>i</sub></i> thì <i>n</i>
<i>m</i>


<i>C</i> là số chẵn.
Từ bổ đề này, ta cũng suy ra rằng <i>n</i>


<i>m</i>


<i>C</i> là chẵn nếu có số nguyên dương <i>k</i> mà 2 |<i>k</i> <i>m</i> nhưng


2 |<i>k</i> <i>n</i> (vì khi đó, <i>m</i> tận cùng bởi <i>k</i> chữ số 0 cịn <i>n</i> thì khơng).


Trở lại bài tốn, ta thấy rằng một xâu nhị phân có <i>a</i> số 0 và <i>b</i> số 1 thì có bậc là <i>C<sub>a b</sub>a</i><sub></sub> 1


(chính là số cách sắp xếp các số 0,1 lên đường thẳng). Do đó, ta cần sắp xếp các số 0,1 phù
hợp sao cho tính từ đầu dãy đến vị trí thứ <i>k</i>, nếu có <i>l</i> số 0 thì <i>l</i>


<i>k</i>
<i>C</i> là lẻ.


<b>Nhận xét 1:</b> 64 số đầu tiên là 0. Thật vậy, nếu trong 64 số đầu tiên có cả 0 lẫn 1 thì <sub>64</sub><i>i</i>
<i>C</i> là
số chẵn, khơng thỏa. Do đó, 64 đầu tiên phải giống nhau, và đều là 0.


<b>Nhận xét 2:</b> 32 số tiếp theo là 1. Tương tự trên, chú ý rằng 5


2 | 96 nên nếu có một số 0 rơi
vào giữa 32 số từ vị trí 6596 thì ta có hai số 65


96



<i>C</i> hoặc 66
96


<i>C</i> đều chẵn.


<b>Nhận xét 3:</b> ba số cuối là 001. Ta chỉ còn ba vị trí với ba số 0, 0,1 và dễ dàng kiểm tra được
rằng nếu xếp theo hai cách kia là 1, 0, 0 thì dẫn đến 65


98


</div>
<span class='text_page_counter'>(7)</span><div class='page_container' data-page=7>

Vậy chỉ có đúng một cách xếp thỏa mãn đề bài.


<b>Bài 8.</b> Tính tổng


2018


2018
1


gcd(2 1, 2<i>a</i> 1)


<i>k</i>
<i>S</i>




<sub></sub>

  và chứng minh rằng 2018 | .<i>S</i>


<b>Lời giải. </b>Ta viết lại thành



2018


gcd( 2018, )
1


2 <i>a</i> 1


<i>k</i>
<i>S</i>




 


<sub></sub>

<sub></sub>  <sub></sub>. Ta thấy rằng:


- Nếu <i>a</i> chẵn và <i>a</i>2018 thì gcd( , 2018)<i>a</i> 2. Có tất cả 1008 số như thế.
- Nếu <i>a</i> lẻ và <i>a</i>1009 thì gcd( , 2018) 1<i>a</i>  . Có tất cả 1008 số như thế.
Từ đó ta tính được


2 1 2018 1009 2018 1009


1008(2 1) 1008(2 1) 2 1 2 1 2 2 2 1008 4


<i>S</i>            


Ta thấy rằng <i>S</i> chẵn nên chỉ cần chứng minh 1009 |<i>S</i> (chú ý rằng 1009 là số nguyên tố).
Theo định lý Fermat nhỏ thì 1009



2 2(mod1009) nên 2018


2 4(mod1009).
Suy ra <i>S</i>   4 0 1008 4  2 20180(mod 2018). Ta có đpcm.


<b>Bài 9. (TPHCM 2017) </b>Cho số nguyên dương <i>n</i> chẵn và một bảng hình chữ nhật <i>n</i> cột, nhiều
hàng được điền số thỏa mãn đồng thời các điều kiện sau:


i) Tổng tất cả các số của mỗi hàng là <i>n</i>.


ii) Mỗi hàng được gán một bậc bằng <i>a</i> và trên mỗi hàng, ta chỉ được điền các số là 0 với <i>a</i>


(khơng có ràng buộc về vị trí của các số trên mỗi hàng). Khơng có hai hàng có bậc bằng nhau.
iii) Khơng thể thêm một hàng mới nào vào bảng mà i), ii) vẫn được thỏa mãn.


Chứng minh rằng nếu trên bảng có lẻ số 0 thì số lẻ lớn nhất trên bảng phải là số chính
phương.


<b>Lời giải. </b>


Giả sử trên một hàng có bậc là <i>a</i>, có tất cả <i>b</i> số <i>a</i> được điền thì tổng các số trên hàng là


<i>n</i><i>ab</i> hay <i>a</i> là ước của <i>n</i>. Theo điều kiện ii, iii) thì số hàng của bảng chính là số ước dương
của <i>n</i>.


Gọi ( ), ( )<i>n f n</i> lần lượt là tổng ước dương của <i>n</i> và số các ước dương của <i>n</i>.
Số các số khác 0 trên một hàng có bậc bằng <i>a</i> là <i>n</i>


<i>a</i> nên số các số bằng 0 trên hàng đó là



.


<i>n</i>
<i>n</i>


</div>
<span class='text_page_counter'>(8)</span><div class='page_container' data-page=8>

Tổng các số 0 trên bảng là



| | |


( ) ( ) ( )


<i>a n</i> <i>a n</i> <i>a n</i>


<i>n</i>


<i>n</i> <i>n a</i> <i>n f n</i> <i>a</i> <i>n f n</i> <i>n</i>


<i>a</i> 


 


        


 


 


.


Do <i>n</i> chẵn nên ( )<i>n</i> phải lẻ. Đặt <i>n</i>2<i>k</i><i>q</i> với <i>q</i> là số lẻ thì <i>q</i> chính là ước lẻ lớn nhất của <i>n</i>,


cũng chính là số lớn nhất có trên bảng. Tổng các ước ( )<i>n</i> của <i>n</i> có dạng ( )


<i>p</i> <i>n</i>
<i>p</i>







.


 Nếu <i>p</i>2 thì rõ ràng (2 )<i>k</i> luôn là số lẻ.


 Nếu <i>p</i>2 thì (<i>p</i>) 1 <i>p</i><i>p</i> là số lẻ nên  phải chẵn.


Từ đó suy ra tất cả các ước nguyên tố lẻ của <i>n</i> đều có số mũ chẵn. Từ đó suy ra <i>q</i> là số chính
phương, ta có đpcm.


<b>Bài 10. (IMO Shortlist 2016) </b>Cho sốngun dương <i>n</i>1. Xét bảng ơ vng kích thước <i>r s</i>


thỏa mãn các điều kiện sau:


i) Tất cả các ước nguyên dương của <i>n</i> đều được điền đúng 1 lần trên bảng và khơng có ơ
trống.


ii) Tổng các số trên mỗi hàng bằng nhau.
iii) Tổng các số trên mỗi cột bằng nhau.


<b>Lời giải. </b>Trước hết, ta có nhận xét rằng <i>r s</i>, 1 vì các ước đều phân biệt.



Tiếp theo, vì trên bảng có số <i>n</i> nên tất cả các tổng của mỗi hàng, mỗi cột đều lớn hơn <i>n</i>.
Giả sử <i>c</i><i>r</i> và gọi <i>a a</i>1, 2,,<i>ar</i> là các ước lớn nhất của hàng 1, 2,,<i>r</i>. Ta giả sử tiếp <i>ar</i> là số


nhỏ nhất trong đó thì <i>ar</i> sẽ khơng vượt quá ước lớn thứ <i>r</i> của <i>n</i> và tất nhiên sẽ không vượt


quá <i>n</i>


<i>r</i> .


Suy ra tất cả các số của cột tương ứng sẽ không vượt quá <i>n</i>


<i>r</i> , có <i>c</i> số như thế nên tổng các số


trên cột đó sẽ khơng vượt q <i>c</i> <i>n</i> <i>r</i> <i>n</i> <i>n</i>


<i>r</i> <i>r</i>


    , mâu thuẫn.


<b>Bài 11. </b>(Korea, 1997) Cho các số nguyên tố <i>p p</i><sub>1</sub>, <sub>2</sub>,,<i>p<sub>k</sub></i> phân biệt và các số tự nhiên bất kì


1, 2, , <i>k</i>


<i>n n</i>  <i>n</i> đều lớn hơn 1. Chứng minh rằng số cặp số ( , )<i>x y</i> khơng có thứ tự, nguyên tố cùng
nhau và thỏa mãn đẳng thức sau 3 3 1 2


1 2
<i>k</i>
<i>n</i>


<i>n</i> <i>n</i>


<i>k</i>


<i>x</i> <i>y</i>  <i>p p</i> <i>p</i> sẽ không vượt quá 1
2 .<i>k</i>
<b>Lời giải. </b>


Ta có


3 3


3 3 <sub>(</sub> <sub>)</sub> <i>x</i> <i>y</i>


<i>x</i> <i>y</i> <i>x</i> <i>y</i>


<i>x</i> <i>y</i>
  


   <sub></sub> <sub></sub>




 




3 3
gcd <i>x</i> <i>y</i>,<i>x</i> <i>y</i> <i>d</i>



<i>x</i> <i>y</i>


  


 


 




 


</div>
<span class='text_page_counter'>(9)</span><div class='page_container' data-page=9>

Ta sẽ chứng minh rằng số cặp ( , )<i>x y</i> khơng có thứ tự thỏa mãn sẽ khơng vượt q số cách
chọn <i>x</i><i>y</i> và


3 3


<i>x</i> <i>y</i>


<i>x</i> <i>y</i>


 . Thật vậy: 3 3 ( , ) 3 ( )3


<i>x</i> <i>y</i> <i>a</i> <i>y</i> <i>a</i> <i>x</i>


<i>a b</i>


<i>x</i> <i>y</i> <i>b</i> <i>x</i> <i>a</i> <i>x</i> <i>b</i>



   


 


 


 


    


 


 .


Ta có


3


3 3 2


( ) 0


3


<i>a</i> <i>b</i>


<i>x</i> <i>a</i> <i>x</i> <i>b</i> <i>x</i> <i>ax</i>


<i>a</i>



       nên phương trình này có khơng q hai nghiệm
là <i>x a</i>0, <i>x</i>0 tương ứng với hai nghiệm của hệ ban đầu là ( ,<i>x a</i>0 <i>x</i>0), (<i>a</i><i>x x</i>0, 0) . Do không có


tính thứ tự nên hai nghiệm này được coi như một nghiệm và nhận xét được chứng minh.
-Tiếp theo, ta sẽ xét hai trường hợp sau:


- Nếu 3

<i>p p p</i>1, 2, 3,...,<i>pk</i>

thì với mỗi


<i>i</i>
<i>n</i>
<i>i</i>


<i>p</i> thì sẽ chỉ thuộc về số


3 3


<i>x</i> <i>y</i>


<i>x</i> <i>y</i>


 hoặc <i>x</i><i>y</i>, tức là có


2 cách chọn và khi đó, số cặp ( , )<i>x y</i> sẽ khơng vượt q 2 .<i>k</i>


- Nếu 3

<i>p p</i>1, 2,...,<i>pk</i>

thì giả sử khơng mất tính tổng qt, có thể giả sử <i>p</i>13. Khi đó, với
 


1 <i>i</i> <i>k</i> thì có 2 cách chọn tương tự như trên cho số <i>ni</i>



<i>i</i>


<i>p</i> và với <i>i</i>1, ta có 4 cách chọn là


1


3<i>n</i> thuộc hoàn toàn về một trong hai số trên hoặc 3 thuộc về một số và <sub>3</sub><i>n</i>11 thuộc về số cịn


lại. Do đó, trong trường hợp này, số cặp ( , )<i>x y</i> sẽ khơng q 1
2<i>k</i> .


Ta có đpcm.


<b>Bài 12. </b>Cho tập hợp <i>A</i>

2 ,2 ,2 ,...,20 1 2 2016

.
1) Hỏi trong <i>A</i> có bao nhiêu số có tận cùng là 4?


2) Hỏi trong <i>A</i> có bao nhiêu số bắt đầu bởi 4, cho biết 22016 bắt đầu bởi số 7 và số này có
tất cả 607 chữ số?


<b>Lời giải. </b>


1) Ta thấy chữ số tận cùng của 2<i>n</i><sub> có chu kỳ là </sub><sub>4</sub><sub>: </sub><sub>2</sub><sub>   </sub><sub>4</sub> <sub>8</sub> <sub>6</sub> <sub>2</sub><sub>. </sub>


Khi đó, số có dạng 24<i>k</i>2


có chữ số tận cùng là 4. Ta tính được có 2016 504


4  số thỏa mãn.



2) Ta xét các chu kỳ thay đổi của chữ số đầu tiên như sau:


1 2 4 8 1,


1 2 4 9 1


1 2 5 1


1 3 6 1


1 3 7 1


</div>
<span class='text_page_counter'>(10)</span><div class='page_container' data-page=10>

Gọi <i>x y</i>, lần lượt là số chu kỳ có độ dài 4, 3.


Sau mỗi chu kỳ, số chữ số tăng lên 1 nên ta có <i>x</i>  <i>y</i> 607 chính là số lượng chữ số.
Ngồi ra, 4<i>x</i> 3<i>y</i> 2017 chính là số lượng lũy thừa.


Ta có hệ phương trình 607


4 3 2017


<i>x</i> <i>y</i>


<i>x</i> <i>y</i>


  



  



 .


Giải hệ này, ta có <i>x</i> 196 nên số lượng các lũy thừa cần tìm là 196.


<b>Bài 13. </b>Hỏi có bao nhiêu số <i>i</i> với 0 <i>i</i> 2015 sao cho <i><sub>C</sub></i><sub>2015</sub><i>i</i> <sub> chia hết cho </sub><sub>3</sub><sub>? </sub>


<b>Lời giải. </b>


Trước hết, ta sẽ đếm các số <i>i</i> mà <i><sub>C</sub></i><sub>2015</sub><i>i</i> <sub> không chia hết cho 3 thông qua định lý Lucas. </sub>


Ta có biểu diễn tam phân của 2015 là: 20152202122(3).


Do 0 <i>i</i> 2015 nên ta xét biểu diễn tam phân của <i>i</i> là


1 2 3 4 5 6 7 (3)


<i>i</i> <i>a a a a a a a</i> với <i>a<sub>i</sub></i> 

0,1,2 ,

<i>i</i>1, 7.
Theo định lý Lucas thì


7
1 2 3 4 5 6


2015 2 2 0 2 1 2 2 (mod 3)


<i>a</i>


<i>a</i> <i>a</i> <i>a</i> <i>a</i> <i>a</i> <i>a</i>


<i>i</i>



<i>C</i> <i>C C C C C C C</i> .
Để số này khơng chia hết cho 3 thì


1 2 4 6 7


0<i>a a a a a</i>, , , , 2, 0<i>a</i><sub>5</sub> 1 và <i>a</i><sub>3</sub>  0.


Số cách chọn các giá trị này là 2 3 5 468.


Vậy số các chỉ số thỏa mãn là 20164861530.


<b>Bài 14. </b>(ELMO 2011) Wanda là một con sâu rất thích ăn các số trên tam giác Pascal. Một ngày
nọ, nó xuất phát từ đỉnh của tam giác, ăn số 0


0 1


<i>C</i>  . Tiếp theo, nó di chuyển đến số nguyên
bên cạnh và tiếp tục ăn số này sao cho:


i. Wanda không được ăn 3 số <i>a b c</i>, , trong tam giác mà <i>a</i>  <i>b</i> <i>c</i>.
ii. Wanda không được di chuyển vào vị trí đã đi qua trước đó.


</div>
<span class='text_page_counter'>(11)</span><div class='page_container' data-page=11>

<b>Lời giải. </b>


Để tránh trường hợp ăn phải các số <i>a b c</i>, , mà <i>a</i>  <i>b</i> <i>c</i>, ta sẽ dùng chiến lược là ăn tất cả các
số lẻ. Điều này có thể thực hiện được theo mơ hình bên dưới (chứng minh chi tiết bằng quy
nạp):


Sử dụng định lý Lucas, ta chứng minh được rằng trong 2<i>n</i> trong đầu tiên, có tất cả 3<i>n</i> số lẻ.
Vì 210 2011211 nên trước hết, Wanda có thể ăn được hết 311 số lẻ trong 2048 dịng đầu


tiên. Từ dịng 2012 2048, có tối đa 20122013...2048 số lẻ.


Vì thế, trong 2011 dịng đầu tiên của tam giác Pascal, có ít nhất


11


3 (20122013...2048)100000 số lẻ.


<i>1</i> <i>8</i> <i>28</i> <i>56</i> <i>70</i> <i>56</i> <i><sub>28</sub></i> <i>8</i> <i>1</i>


<i>1</i>


<i>7</i> <i>21</i> <i>35</i> <i>35</i> <i>21</i> <i>7</i> <i>1</i>


<i>1</i>


<i>6</i> <i>15</i> <i>20</i> <i>15</i> <i>6</i>


<i>1</i>


<i>1</i>


<i>5</i> <i>10</i> <i>10</i> <i>5</i>


<i>1</i>


<i>1</i>


<i>4</i> <i>6</i> <i>4</i>



<i>1</i>


<i>1</i>
<i>1</i>


<i>3</i>
<i>3</i>


<i>1</i>


<i>1</i>
<i>1</i>


<i>2</i> <i>1</i>


</div>

<!--links-->

×